Klausur Physik 2 am 11.3.1998 1. Wärmekraftmaschine 2

Werbung
Name, Matrikelnummer:
1. Wärmekraftmaschine
Klausur Physik 2 am 11.3.1998
Eine zyklisch und reversibel arbeitende Wärmekraftmaschine mit Wasserstoff als Arbeitsgas
habe den Wirkungsgrad 0,19. Das kältere Wärmereservoir habe Zimmertemperatur (20o C).
Fachbereich Elektrotechnik, Fachhochschule Bochum
a. Wie groß ist die Temperatur des wärmeren Reservoirs?
Zugelassene Hilfsmittel: Zusammenfassung der Vorlesung (maximal 6 Seiten A4), Beiblätter
zur Vorlesung Physik 2 im WS 97, Formelsammlung, Taschenrechner
(5 Punkte)
Dauer: 2 Stunden
b. Kann der Wirkungsgrad, bzw. die Leistungszahl von Wärmepumpen größer als 1 sein
(Begründung)? Werden der 1. oder 2. Hauptsatz der Thermodynamik evtl. verletzt?
(5 Punkte)
Maximal erreichbare Punktezahl: 100. Bestanden hat, wer mindestens 50 Punkte erreicht.
2. Wärmeleitung
Bitte verwenden Sie bei Berechnungen zunächst die gegebenen symbolischen Größen und
setzten Sie erst am Schluß die Zahlenwerte (mit Einheiten!) ein.
Zwei unendlich große Wärmereservoirs der Temperaturen T1 = 200o C und T2 = 80o C sind
durch einen Stab der Länge l = 30 cm und Durchmesser D = 1 cm aus Kupfer
(Wärmeleitfähigkeit λ = 395 W/(mK)) verbunden.
Bitte kennzeichnen Sie dieses Blatt und alle weiteren, die Sie verwenden, mit Ihrem Namen
und Ihrer Matrikelnummer.
AUFGABE
1.a
1.b
2.a
2.b
2.c
3.a
3.b
3.c
3.d
4.a
4.b
4.c
5.a
5.b
5.c
6.a
6.b
Summe
Cu
MÖGLICHE
ERREICHTE
PUNKTZAHL PUNKTZAHL
5
5
5
10
5
5
10
5
5
5
5
5
10
5
5
5
5
100
T1
T2
D
l
a. Wie groß ist der thermische Widerstand des Stabs?
(5 Punkte)
b. Welche Wärmemenge fließt in 20 Minuten durch den Stab?
(10 Punkte)
c. Wenn man diese Wärmemenge verwendet, um einen Liter Wasser (1 kg, spezifische
Wärmekapazität c = 4,18 kJ/(kg.K)) zu erwärmen, um wieviel Grad steigt die Temperatur
des Wassers?
(5 Punkte)
Seite 1 von 9
Seite 2 von 9
3. Federpendel
5. Doppelspalt
Ein Federpendel sei gebildet durch eine Masse m = 0,5 kg und eine Feder der Federkonstante
D = 177 N/m. Es werde durch eine periodische Kraft der Amplitude F0 = 2 N zum Schwingen
angeregt. Bei der Anregungsfrequenz 0 beträgt die Auslenkung des Pendels x0. Bei einer
Frequenz von fR = 2,6 Hz erreicht die Auslenkung ein Maximum.
Ein Doppelspalt mit Spaltabstand d = 0,1 mm werde mit kohärentem Licht durchstrahlt. Der
Spalt habe vom Projektionsschirm einen Abstand von l1 = 5 m. Dort beobachtet man im
Abstand von l2 = 3,15 cm vom Mittelstrahl ein erstes Beugungsmaximum. Die Spaltgröße ist
zu vernachlässigen.
l3
d
Lichtquelle
Feder mit Federkonstante D
l1
Doppelspalt
Projektionsschirm
a. Wie groß ist die Wellenlänge λ des verwendeten Lichts?
periodische Kraft F0sin(ωt)
l2
(10 Punkte)
b. In welchem Abstand l3 vom Mittelstrahl beobachtet man das zweite Beugungsmaximum?
(5 Punkte)
a. Skizzieren Sie qualitativ die Amplitude x der erzwungenen Schwingung über der
Anregungsfrequenz.
(5 Punkte)
b. Berechnen Sie die Abklingkonstante λ und die Dämpfungskonstante r. (10 Punkte)
c. Wie groß ist die Güte Q des Oszillators?
(5 Punkte)
d. Wie groß ist die Gesamtenergie Eges des Federpendels in Resonanz? (5 Punkte)
4. Wellen
a. Erläutern Sie am Beispiel elastischer Wellen die drei zu unterscheidenden
Geschwindigkeiten. (5 Punkte)
c. Was für eine Lichtquelle verwendet man zweckmäßigerweise für einen solchen Versuch?
(5 Punkte)
6. Geometrische Optik
Betrachten Sie die Abbildung eines Objekts mit einer Sammellinse der Brennweite f.
a. Skizzieren Sie die Abhängigkeit der Vergrößerung V von der Gegenstandsweite g.
(5 Punkte)
b. Bei welcher Gegenstandsweite ist die Vergrößerung 1 (eins) ? (5 Punkte)
b. Betrachten Sie Wasserwellen auf dem Kemnader See. Handelt es sich um Longitudinaloder Transversalwellen? Warum?
(5 Punkte)
c. Welche Winkel bilden bei der Ausbreitung elekromagnetischer Wellen im Vakuum
elektrisches und magnetisches Feld mit der Ausbreitungsrichtung und elektrisches und
magnetisches Feld untereinander?
(5 Punkte)
Seite 3 von 9
Seite 4 von 9
Seite 5 von 9
Seite 6 von 9
Seite 7 von 9
Seite 8 von 9
Name, Matrikelnummer:
Klausur Physik 2 am 16.9.1998
Fachbereich Elektrotechnik, Fachhochschule Bochum
Zugelassene Hilfsmittel: Zusammenfassung der Vorlesung (maximal 6 Seiten A4), Beiblätter
zur Vorlesung Physik 2 im WS 97, Taschenrechner
Dauer: 2 Stunden
Maximal erreichbare Punktezahl: 100. Bestanden hat, wer mindestens 50 Punkte erreicht.
Bitte verwenden Sie bei Berechnungen zunächst die gegebenen symbolischen Größen und
setzten Sie erst am Schluß die Zahlenwerte (mit Einheiten!) ein.
Bitte kennzeichnen Sie dieses Blatt und alle weiteren, die Sie verwenden, mit Ihrem Namen
und Ihrer Matrikelnummer.
AUFGABE
1.a
1.b
2.a
2.b
3.a
3.b
3.c
4.a
4.b
5.a
5.b
5.c
5.d
6.a
6.b
6.c
Summe
MÖGLICHE
ERREICHTE
PUNKTZAHL PUNKTZAHL
10
15
5
5
5
5
5
10
5
5
10
3
3
8
3
3
100
Seite 9 von 9
Seite 1 von 7
1. Federpendel
5. Kreisprozeß des idealen Gases
Ein gedämpftes Federpendel (Masse 50 g) wird zu erzwungenen Schwingungen angeregt. Bei
der Frequenz f = 2700 Hz findet man eine Phasendifferenz von –170o zwischen Auslenkung
und anregender Kraft. Die Eigenfrequenz des ungedämpften Federpendels beträgt 1600 Hz.
Ein ideales Gas durchläuft folgenden Kreisprozess:
a. Wie groß ist die Dämpfungskonstante r des gedämpften Federpendels?
b. Um welchen Betrag muss man die Masse des Federpendels verringern, um bei der
gleichen Frequenz von 2700 Hz eine Phasendifferenz von –160o zu erreichen? Die
Dämpfung bleibe unverändert.
2. Absorption von Wellen
-
isobare Expansion
isochore Erwärmung
isobare Kompression
isochore Abkühlung
a. Skizzieren Sie den Kreisprozess im p-V-Diagramm.
b. Berechnen Sie die am System geleistete Arbeit für jeden Prozessschritt und für den
gesamten Kreisprozess. Um welchen Typ thermodynamischer Maschinen handelt es sich?
c. Berechnen Sie für die isochoren Prozessschritte die Änderung der inneren Energie.
d. Von welchen Zustandsvariablen hängt die innere Energie beim idealen Gas ab?
Eine Lichtwelle verliert beim Durchtritt durch eine 1 cm dicke Glasscheibe 15% ihrer
Energie.
6. Strömung idealer Flüssigkeiten
a. Wie groß ist der Absorptionskoeffizient?
b. Wieviel Prozent der Intensität (Energie) verliert die Welle beim Durchtritt durch eine 1 m
dicke Scheibe des gleichen Materials?
Als ideale Flüssigkeit angenommenes Wasser durchströmt die folgende Anordnung aus
Rohren:
3. Brechung und Totalreflexion
Der Grenzwinkel für Totalreflexion beim Übergang von einem optisch dichteren Medium zur
Luft beträgt 40o. Für Luft sei eine Brechzahl von 1 angenommen.
a. Wie groß ist die Phasengeschwindigkeit in diesem Medium?
b. Gibt es solche Materialien in Wirklichkeit? Was könnte es sein?
c. Ein Strahl treffe unter einem Winkel von 20o zum Lot aus dem Medium auf die
Grenzfläche zur Luft. Unter welchem Winkel zum Lot verlässt der Strahl die
Grenzfläche? Skizzieren Sie die Anordnung.
4. Wärmeleitung
Eine Tasse enthält 200 ml Kaffee mit einer Temperatur von 65o C. Eine Studentin gießt dort
20 ml Milch mit einer Temperatur von 20o C hinein. Kaffee habe etwa die Dichte und
spezifische Wärmekapazität von Wasser: 4,18 kJ/(kg.K), Milch habe eine Dichte von 1,02
g/cm3 und eine spezifische Wärmekapazität von 3,8 kJ/(kg.K). Die Wärmekapazität der
Gefäße sowie Wärmeleitung seien vernachlässigt.
Querschnitt A1,
v1, p1
Querschnitt A2,
v2, p2
Strömungsrichtung
Die Höhe s sei als groß gegenüber den Rohrradien angenommen. Der Querschnitt A1 beträgt
4,5 cm2, die Geschwindigkeit v1 30 cm/s und der statische Druck p1 105 Pa. Der Querschnitt
A2 beträgt 11 cm2 und die Höhendifferenz s 0,5 m. Die Dichte des Wassers sei konstant.
a. Wie groß sind die Geschwindigkeit v2 und der Druck p2?
b. Spielt die Strömungsrichtung eine Rolle?
c. Was ändert sich, wenn anstatt des Wassers ein Gas strömt?
a. Welche Temperatur nimmt der Milchkaffee an?
b. Berücksichtigt man die Wärmeleitung, so ergibt sich die alte Frage, ob der Kaffee wärmer
bleibt, wenn man erst die Milch hineingießt und dann 10 Minuten wartet, oder erst 10
Minuten wartet und dann die Milch hineingießt. Nehmen Sie dazu physikalisch fundiert
(qualitativ) Stellung.
Seite 2 von 7
s
Seite 3 von 7
Seite 4 von 7
Seite 5 von 7
Seite 6 von 7
Seite 7 von 7
Name, Matrikelnummer:
1. Gedämpfte Schwingung
Gedämpfte Schwingung
Auslenkung [cm]
Klausur Physik 2 am 10.3.99
Fachbereich Elektrotechnik, Fachhochschule Bochum
Zugelassene Hilfsmittel: Zusammenfassung der Vorlesung (maximal 6 Seiten A4), Beiblätter
zur Vorlesung Physik 2 im WS 98/99 oder WS 97/98, Taschenrechner
3
x1
2
x2
1
0
Dauer: 2 Stunden
Maximal erreichbare Punktezahl: 100. Bestanden hat, wer mindestens 50 Punkte erreicht.
-1
0
t1
0,5
1,5 t2
2
Zeit [s]
-2
Bitte beginnen Sie die Lösung der Aufgabe unbedingt auf dem betreffenden
Aufgabenblatt! Falls Sie weitere Blätter benötigen, müssen diese unbedingt deutlich mit
der Aufgabennummer gekennzeichnet sein.
1
-3
Bitte verwenden Sie bei Berechnungen zunächst die gegebenen symbolischen Größen und
setzten Sie erst am Schluß die Zahlenwerte (mit Einheiten!) ein.
t1 = 0,175 s
x1 = 2,29 cm
Bitte kennzeichnen Sie dieses Blatt und alle weiteren, die Sie verwenden, mit Ihrem Namen
und Ihrer Matrikelnummer.
Gegeben ist die oben gezeichnete gedämpfte Schwingung mit Maxima bei t1 und t2 und den
dazugehörigen Auslenkungen x1 und x2.
AUFGABE
a. Wie groß sind die Kreisfrequenz und die Abklingkonstante dieser gedämpften
Schwingung?
1.a
1.b
1.c
1.d
2.a
2.b
2.c
3.a
3.b
3.c
3.d
4.a
4.b
4.c
5.b
5.c
5.d
Summe
MÖGLICHE
ERREICHTE
PUNKTZAHL PUNKTZAHL
10
4
3
3
7
6
7
4
10
3
3
7
7
6
7
7
6
100
t2 = 1,575 s
x2 = 1,14 cm
b. Wie lautet die Auslenkung x(t) dieser Schwingung in Abhängigkeit der gegebenen
Größen und der Zeit?
c. Wie groß wäre die Frequenz (in Hz), mit der der ungedämpfte Oszillator schwingen
würde?
d. Wie groß ist die potentielle Energie zur Zeit t = 0 ?
Seite 1 von 11
Seite 2 von 11
2. Akustik
3. Planparallele Platte
Eine punktförmige Schallquelle strahlt gleichförmig in alle Richtungen. Im Abstand von 5 m
misst man einen Schallintensitätspegel von 92 db.
Ein Lichtstrahl trifft unter einem Winkel von 32o zur Oberfläche auf eine Glasplatte der Dicke
5 cm mit parallelen Flächen (Brechzahl 1,58). Außerhalb der Platte sei die Brechzahl 1.
a. Welchen Schallintensitätspegel misst man in einem Abstand von 12 m?
Hinweis zu a. und b.: Nehmen Sie für die Berechnung der Schallintensität die Fläche einer
Kugel mit Radius R. Damit erfassen Sie die vollständige Leistung der Schallquelle.
b. Die Schallquelle werde nun vollständig abgeschirmt, so dass kein Schall mehr nach außen
dringt. Welche Energie nimmt die Abschirmung in 10 Minuten auf, wenn sie den Schall
vollständig absorbiert?
c. Eine Schallquelle erzeugt an einem Ort einen Schallintensitätspegel von 72 db, eine
zweite (allein) von 65 db. Welcher Schallintensitätspegel ergibt sich, wenn beide
Schallquellen eingeschaltet sind?
n=1
32o
n = 1,58
5 cm
n=1
Hinweis: Interferenz spielt keine Rolle, die Schallintensitäten addieren sich.
a. Skizzieren Sie präzise den Weg des Strahls in der Platte und hinter der Platte.
b. Welchen Abstand hat der Strahl hinter der Platte von der gestrichelten Linie, d.h. von der
geradlinigen Fortsetzung des ankommenden Strahls?
c. An der Ober- und Unterseite der Platte kann der Strahl auch reflektiert werden. Welche
Reflexionswinkel (zum Lot) ergeben sich in beiden Fällen?
d. Wie groß müsste der Einfallswinkel (zur Oberfläche) beim Eintritt in die Platte sein, damit
an einer der Flächen Totalreflexion auftritt? An welcher Fläche würde dann Totalreflexion
auftreten?
Seite 3 von 11
Seite 4 von 11
4. Videokamera
5. Widerstand im Wasser
Eine Videokamera ist mit einem Objektiv ausgestattet, das man zwischen den Brennweiten
35 mm und 70 mm umschalten kann. Der maximale Abstand zwischen CCD-Chip und Linse
beträgt bei 35 mm Brennweite 4 cm, und bei 70 mm Brennweite 7,1 cm.
Ein elektrischer Widerstand liegt in 200 ml destilliertem Wasser mit einer Temperatur von
20o C (cWasser = 4,18 kJ/(kg.K)). Destilliertes Wasser ist ein Isolator. Der Widerstand wird
mit einer Spannungsquelle verbunden, die 10 V liefert. Nach 6 Minuten beginnt das Wasser
zu sieden (Normaldruck). Wärmeverluste seien vernachlässigt, die Wärmeleitung sei ideal.
a. Wie dicht kann man mit der Linse an ein Objekt heran gehen, um noch ein scharfes Bild
zu erhalten (ohne Berücksichtigung der Tiefenschärfe)? Bei welcher Brennweite ergibt
sich der kleinste Abstand?
a. Wie groß ist der Widerstand (in Ω)?
b. Wie ist die Vergrößerung (Bildgröße zu Gegenstandsgröße) bei Verwendung der 70 mm Linse und dem maximalen Abstand zwischen Linse und CCD-Chip?
b. Wenn man das Wasser (200 ml, 20o C) anstatt mit dem Widerstand mit einer Heizplatte
mit 500 W Leistung erwärmt, wann ist es vollständig verdampft (Verdampfungswärme
von Wasser: 2256 kJ/kg)? Wärmeverluste seien vernachlässigt, die Wärmeleitung sei
ideal.
c. Skizzieren Sie für eine Brennweite von 35 mm die Bildweite in Abhängigkeit von der
Gegenstandsweite.
c. Wie groß ist im Aufgabenteil b. der Wärmestrom? Welchen thermischen Widerstand hat
das Wassergefäß zu Beginn des Aufheizens, wenn die Herdplatte eine Temperatur von
300o C hat?
Seite 5 von 11
Lösung zur Aufgabe 1:
Seite 6 von 11
Lösung zu Aufgabe 2:
Seite 7 von 11
Seite 8 von 11
Lösung zu Aufgabe 3:
Lösung zu Aufgabe 4:
Seite 9 von 11
Lösung zu Aufgabe 5:
Seite 10 von 11
Name, Matrikelnummer:
Klausur Physik 2 am 22.9.99
Fachbereich Elektrotechnik, Fachhochschule Bochum
Zugelassene Hilfsmittel: Zusammenfassung der Vorlesung (maximal 6 Seiten A4), Beiblätter
zur Vorlesung Physik 2 im WS 98 oder WS 97, Taschenrechner
Dauer: 2 Stunden
Maximal erreichbare Punktezahl: 100. Bestanden hat, wer mindestens 50 Punkte erreicht.
Bitte beginnen Sie die Lösung der Aufgabe unbedingt auf dem betreffenden
Aufgabenblatt! Falls Sie weitere Blätter benötigen, müssen diese unbedingt deutlich mit
der Aufgabennummer gekennzeichnet sein.
Bitte verwenden Sie bei Berechnungen zunächst die gegebenen symbolischen Größen und
setzten Sie erst am Schluß die Zahlenwerte (mit Einheiten!) ein.
Bitte kennzeichnen Sie dieses Blatt und alle weiteren, die Sie verwenden, mit Ihrem Namen
und Ihrer Matrikelnummer.
AUFGABE
1.a
1.b
1.c
1.d
2.a
2.b
2.c
2.d
2.e
3.a
3.b
3.c
4.a
4.b
4.c
5.a
5.b
5.c
Summe
Seite 11 von 11
MÖGLICHE
ERREICHTE
PUNKTZAHL PUNKTZAHL
5
5
5
5
8
3
3
3
3
8
8
4
7
7
6
5
8
7
100
Seite 1 von 11
2. Eindimensionale akustische Welle
1. Ungedämpfte Schwingung mit zwei Massen
Eine senkrecht stehende Feder wird mit einem Gewicht der Masse 20,5 kg belastet und senkt
sich unter dieser Belastung um 9,5 cm. Zum Zeitpunkt t = 0 wird an diesem Ort ein zweiter
Körper der Masse 15,3 kg auf dem ersten befestigt und das System dann losgelassen. Es
kommt zu Schwingungen. Die Masse der Feder und die Dämpfung seien vernachlässigt.
a. Bei welcher Auslenkung (zur unbelasteten Feder) ist die Gleichgewichtslage des Systems,
d.h. die Gesamtkraft null?
Eine Schallwelle der Frequenz 677 Hz breitet sich in Luft geradlinig aus. Dabei schwingen
die Luftmoleküle mit einer Amplitude von 0,21 mm. Die Schallgeschwindigkeit beträgt
340 m/s.
a. Wie lautet die Gleichung, die die Auslenkung der Luftmoleküle in Abhängigkeit von Ort
und Zeit beschreibt (also die Lösung der Wellengleichung), wenn zum Zeitpunkt t = 0 s
die Auslenkung am Ort x = 0 m null ist? Wie groß sind Wellenlänge, Kreisfrequenz und
Wellenzahl?
b. Skizzieren Sie die Momentaufnahme der Auslenkung zum Zeitpunkt t = 0 s. Zeichnen Sie
die Wellenlänge ein.
b. Wie groß ist die Amplitude der Schwingung?
c. Wie groß ist die Kreisfrequenz ω0, wie groß ist die Eigenfrequenz f0?
d. Welche Geschwindigkeit haben die Körper zur Zeit t = 4 s?
c. Skizzieren Sie die Geschwindigkeit der Luftmoleküle am Ort x = 0 m über der Zeit. Wie
groß ist die maximale Geschwindigkeit?
d. Wie würde sich bei gleicher Frequenz die Wellenlänge ändern, wenn die Temperatur um
23o C zunimmt?
e. Kann man den Ton von 677 Hz, ausreichende Lautstärke vorausgesetzt, hören? Welche
Frequenzen kann das menschliche Gehör wahrnehmen?
Seite 2 von 11
Seite 3 von 11
3. Interferenz von Natrium-Licht
4. Polarisationsbrillen
Kohärentes Licht aus einer Natrium-Dampflampe fällt auf ein Beugungsgitter. Das NatriumLicht enthält zwei dicht benachbarte Spektrallinien, die sich nur um 0,6 nm Wellenlänge
unterscheiden. Das erste Beugungsmaximum für die eine Spektrallinie liegt bei 6,765o, für die
andere Spektrallinie bei 6,772o.
In der Vergangenheit sind Versuche zur Darstellung dreidimensionaler Filme mit Hilfe von
Polarisationsbrillen gemacht worden. Dabei erhält jedes Auge einen Polarisationsfilter, die
Polarisationsebenen stehen senkrecht aufeinander. (Das dreidimensionale Sehen beruht
darauf, dass die beiden Augen des Menschen unterschiedliche Informationen erhalten. Das
Gehirn errechnet daraus das dreidimensionale Bild.)
a. Wie groß ist der Spaltabstand des Beugungsgitters?
b. Wie groß ist der Frequenzunterschied zwischen beiden Spektrallinien im Vakuum
(c0 = 2,99.108 m/s)?
c. Je größer die Beugungsordnung, und damit der Beugungswinkel wird, um so geringer ist
die Intensität der Beugungsmaxima. Warum?
a. Wie könnte eine Anordnung zur Wiedergabe dreidimensionaler Filme mit
Polarisationsbrillen aussehen? Skizzieren Sie!
Unpolarisiertes Licht fällt auf ein Polarisationsfilter (Polarisator). Hinter dem Polarisator
befindet sich ein zweites Polarisationsfilter (Analysator). Die Polarisationsrichtungen beider
Filter sind gleich.
b. Um welchen Winkel muss die Polarisationsrichtung des Analysators gegenüber dem
Polarisator gedreht werden, damit die Intensität um 95% gegenüber dem unpolarisierten
Licht abnimmt?
c. Wozu werden in der Fotografie Polarisationsfilter hauptsächlich verwendet?
Seite 4 von 11
Seite 5 von 11
5. Kreisprozess mit drei Punkten
Lösung zu Aufgabe 1:
Mit einem idealen Gas der konstanten Stoffmenge n wird ein Kreisprozess ausgeführt, der
sich aus folgenden Zustandsänderungen zusammensetzt, die in der angegebenen Reihenfolge
durchlaufen werden. Ausgangspunkt ist (V1, p1, T1).
1. isobare Ausdehnung auf Volumen V2 bei Temperatur T2
2. isotherme Zustandsänderung
3. isochore Zustandsänderung
a. Stellen Sie den Kreisprozess im p-V-Diagramm dar und bezeichnen Sie die relevanten
Größen.
b. Berechnen Sie die während eines Kreisprozess-Durchlaufs am System geleistete Arbeit in
Abhängigkeit der gegebenen Größen p1, V1, V2, T1, T2 und n. Die Prozesse seien
reversibel. Leistet das Gas Arbeit?
c. Berechnen Sie für den isochoren Prozessschritt die Änderung der inneren Energie.
Seite 6 von 11
Lösung zu Aufgabe 2:
Seite 7 von 11
Lösung zu Aufgabe 3:
Seite 8 von 11
Seite 9 von 11
Lösung zu Aufgabe 4:
Lösung zu Aufgabe 5:
Seite 10 von 11
Name, Matrikelnummer:
Seite 11 von 11
1. Polarisation duch Reflexion
Reflektiertes Licht ist vollständig polarisiert, wenn es einen Winkel von 90o mit dem
gebrochenen Strahl bildet.
Klausur Physik 2 am 15.3.00
Fachbereich Elektrotechnik und Informatik
Zugelassene Hilfsmittel: Zusammenfassung der Vorlesung (maximal 6 Seiten A4 einseitig
oder 3 Seiten A4 doppelseitig), Beiblätter zur Vorlesung Physik 2 im WS99, WS 98 oder WS
97, Taschenrechner (ohne drahtlose Übertragung mit einer Reichweite von größer als 30 cm
wie Funkmodem, IR-Sender)
Dauer: 2 Stunden
a. Unter welchem Winkel zum Lot muss ein Strahl aus der Luft (n = 1) auf eine
Glasoberfläche mit n = 1,52 treffen, damit das reflektierte Licht vollständig polarisiert ist?
(Hinweis: sin(90o - α) = cosα)
b. In den reflektierten, vollständig polarisierten Strahl wird jetzt ein Polarisationsfilter
gehalten. Unter welchem Winkel zur Polarisationsrichtung des reflektierten Strahls muss
die Durchlassrichtung des Polarisationsfilters angeordnet werden, damit der reflektierte
Strahl beim Durchgang durch das Polarisationsfilter 90% seiner Intensität verliert?
c. Skizzieren Sie die Anordnung.
Maximal erreichbare Punktezahl: 100. Bestanden hat, wer mindestens 50 Punkte erreicht.
Bitte beginnen Sie die Lösung der Aufgabe unbedingt auf dem betreffenden
Aufgabenblatt! Falls Sie weitere Blätter benötigen, müssen diese unbedingt deutlich mit
der Aufgabennummer gekennzeichnet sein.
Bitte verwenden Sie bei Berechnungen zunächst die gegebenen symbolischen Größen und
setzten Sie erst am Schluß die Zahlenwerte (mit Einheiten!) ein.
Bitte kennzeichnen Sie dieses Blatt und alle weiteren, die Sie verwenden, mit Ihrem Namen
und Ihrer Matrikelnummer.
AUFGABE
1.a
1.b
1.c
2.a
2.b
2.c
3.a
3.b
3.c
4.a
4.b
4.c
5.a
5.b
5.c
Summe
MÖGLICHE
ERREICHTE
PUNKTZAHL PUNKTZAHL
10
7
3
6
10
4
3
7
10
8
7
5
10
5
5
100
Seite 1 von 11
Seite 2 von 11
3. Ausdehnung von Aluminium und Stahl
2. Absorption verschiedener Frequenzen
Ein Gemisch aus Licht zweier Frequenzen 1 und 2 durchläuft ein absorbierendes Medium, das
für die beiden Frequenzen unterschiedliche Absorptionskoeffizienten hat. Der
Absorptionskoeffizient für die Frequenz 1 beträgt α1 = 2 1/m. Das Verhältnis der Intensitäten
beträgt beim Eintritt in das Medium I1/I2 = 3. Nach Durchlaufen einer Strecke von 79 cm
beträgt das Verhältnis 0,8.
a. Schreiben Sie die in der Aufgabe genannten Fakten in Form von Gleichungen auf.
Schätzen Sie ab, ob α2 größer oder kleiner als α1 ist. Begründen Sie Ihre Abschätzung.
Ein Stab bestehe aus Aluminium (Längenausdehnungskoeffizient: 24.10-6 1/K), ein anderer
aus V2A-Stahl (Längenausdehnungskoeffizient: 16.10-6 1/K).
a. Woher kommt es, dass sich Körper in der Regel bei höheren Temperaturen ausdehnen?
b. Geben Sie die Abhängigkeit der Länge der Stäbe von der Temperatur in Grad Celcius an,
wenn die Längen bei 0o C gegeben sind.
c. Wie lang müssen die Stäbe bei 0o C sein, damit der Stahlstab bei beliebiger
Temperaturänderung immer 10 cm länger als der Aluminiumstab ist?
b. Wie groß ist α2 ?
c. Begründen Sie, warum es in einem Medium unterschiedliche Absorptionskoeffizienten für
verschiedene Frequenzen geben kann.
Seite 3 von 11
Seite 4 von 11
4. Verdunstung von Wasser
5. Auftrieb eines Gasballons
900 g Eis bei -10 oC werden bei Normaldruck so stark erhitzt, dass daraus 130o C heißer
Wasserdampf wird. (spez.Wärmekapazität von Eis: 2,09 kJ/(kgK), spezifische
Schmelzwärme: 334 kJ/kg, spez. Wärmekapazität von Wasser: 4,18 kJ/(kgK), spez.
Verdampfungswärme: 2256 kJ/kg, spez. Wärmekapazität von Dampf: 2,0 kJ/(kgK))
Auf einen Körper wirkt in Flüssigkeiten und Gasen eine Auftriebskraft, die gleich der
Gewichtskraft des vom Körper verdrängten Mediums ist.
a. Wie groß ist die insgesamt zugeführte Wärme?
b. Welches Volumen nimmt der Dampf bei Normaldruck (1013 hPa) ein? (Molmasse von
Wasser: 18 g/mol, Gaskonstante R = 8,314 J/(molK))
c. Skizzieren Sie die Temperatur über der zugeführten Wärme.
Ein Ballon hat eine unelastische, unten offene Hülle vom Volumen VB = 4,8.103 m3 und wird
bei der Temperatur 20 oC und dem Druck 104 kPa vollständig mit Wasserstoff gefüllt.
Ballonhülle und Nutzlast (ohne Gas) haben zusammen die Masse 3 t. Wasserstoff und Luft
können als ideales Gas betrachtet werden.
a. Wie groß ist die Masse des Wasserstoffs im Ballon, wenn die Dichte bei
Normalbedingungen p0 = 101 kPa und bei 0 oC 0,09 kg/m3 beträgt? Beachten Sie, dass
der Ballon nicht unter Normalbedingungen befüllt wurde!
b. Wie groß ist die Masse der vom Ballon verdrängten Luft, wenn diese unter
Normalbedingungen eine Dichte von 1,29 kg/m3 hat? Beachten Sie auch hier, dass der
Aufstieg des Ballons nicht unter Normalbedingungen erfolgt.
c. Wie groß ist die Gesamtkraft, die im Moment des Aufsteigens auf den Ballon wirkt?
Seite 5 von 11
Seite 6 von 11
Lösung zu Aufgabe 1:
Lösung zu Aufgabe 2:
Seite 7 von 11
Seite 8 von 11
Lösung zu Aufgabe 3:
Lösung zu Aufgabe 4:
Seite 9 von 11
Seite 10 von 11
Lösung zu Aufgabe 5:
Name, Matrikelnummer:
Klausur Physik 2 am 21.9.2000
Fachbereich Elektrotechnik und Informatik
Zugelassene Hilfsmittel: Zusammenfassung der Vorlesung (maximal 6 Seiten A4 einseitig
oder 3 Seiten A4 doppelseitig), Beiblätter zur Vorlesung Physik 2 im WS99, WS 98 oder WS
97, Taschenrechner (ohne drahtlose Übertragung mit einer Reichweite von größer als 30 cm
wie Funkmodem, IR-Sender)
Dauer: 2 Stunden
Maximal erreichbare Punktezahl: 100. Bestanden hat, wer mindestens 50 Punkte erreicht.
Bitte beginnen Sie die Lösung der Aufgabe unbedingt auf dem betreffenden
Aufgabenblatt! Falls Sie weitere Blätter benötigen, müssen diese unbedingt deutlich mit
der Aufgabennummer gekennzeichnet sein.
Bitte kennzeichnen Sie dieses Blatt und alle weiteren, die Sie verwenden, mit Ihrem Namen
und Ihrer Matrikelnummer.
AUFGABE
1.a
1.b
1.c
2.a
2.b
2.c
3.a
3.b
3.c
4.a
4.b
5.a
5.b
5.c
Summe
MÖGLICHE
ERREICHTE
PUNKTZAHL PUNKTZAHL
6
10
4
6
10
4
6
4
10
10
10
8
8
4
100
Seite 11 von 11
Seite 1 von 11
1. Feder mit drei Gewichten
2. Rückspiegel
An eine senkrecht aufgehängte Spiralfeder der Federkonstante D werden drei Gewichte
gehängt, von denen zwei gleich sind. Unter dieser Last dehnt sich die Feder um die Strecke
x1. Lenkt man die Gewichte aus, so schwingt das System mit der Frequenz f3. Nun entfernt
man eines der beiden gleichen Gewichte. Bei Auslenkung schwingt das System jetzt mit der
Frequenz f2. Die Dämpfung werde vernachlässigt, ebenso die Masse der Feder.
a. Fassen Sie die oben gemachten Aussagen in Gleichungen. Vergessen Sie dabei nicht das
Hook'sche Gesetz.
Eine Autofahrerin betrachtet im Rückspiegel ein hinter ihr fahrendes Auto. Bitte entnehmen
Sie die Maße und Näherungen der unten stehenden Skizze.
a = 0,5 m
h = 0,35 m
ϕ = 18o
α und ψ sind Hilfsgrößen
Rückspiegel
.
b. Berechnen Sie die drei beteiligten Massen in Abhängigkeit von g, f3, f2, D und x1.
c. Betrachten Sie ein schwingendes, ungedämpftes System. Wie lautet dabei (in Worten) der
Impulsatz?
α
hinterher fahrendes Auto
ψ
Fahrerin
l
ϕ
h
a
a. Berechnen Sie den Winkel α des betrachteten Strahls zum Lot auf dem Rückspiegel in
Abhängigkeit von a, h und ϕ. Hinweise: Beachten Sie, dass die Summe aus α, ψ und ϕ
den rechten Winkel ergibt, betrachten Sie ferner das Dreieck mit den Seiten a und h.
b. Berechnen Sie den Abstand l des hinterher fahrenden Autos von der Fahrerin. Hinweis:
Betrachten Sie das Dreieck mit den Seiten l + a und h.
c. Welcher (kleine) Effekt müsste eigentlich noch berücksichtigt werden, wenn man
bedenkt, dass der Strahl schräg durch die Heckscheibe hindurchtritt?
Seite 2 von 11
Seite 3 von 11
3. Hoher und tiefer Ton
4. Erhitzter Autoreifen
Ein Messgerät registriert bei zwei aufeinander folgenden Tönen von 63 Hz und 4 kHz jeweils
die gleiche Intensität von 10-4 W/m2.
a. Wie groß sind die zugehörigen Schallintensitätspegel? (I0 = 10-12 W/m2) Welche
Bedeutung hat I0?
b. Was ist der Unterschied zwischen dem Schallintensitätspegel und dem Schalldruckpegel?
c. Im vorliegenden Fall seien Schallintensitäts- und Schalldruckpegel gleich. Schätzen Sie
anhand des unten stehenden Diagramms die Lautstärken der beiden Töne ab. Erläutern Sie
Ihr Vorgehen.
Nach einer langen Autofahrt ist der Reifen (einschließlich Luft) eines Autos auf 65o C
aufgeheizt. Der Fahrer misst an der Tankstelle einen zu hohen Reifendruck, und lässt soviel
Luft ab (was man nach einer langen Fahrt nie tun sollte), bis der Solldruck von 2,4 bar
Überdruck (über Normaldruck von 1 bar = 105 Pa) erreicht ist. Am nächsten Morgen ist der
Reifen (einschließlich Luft) auf 19o C abgekühlt. (Luft sei als ideales Gas betrachtet, das
Volumen des Reifens ist unabhängig von Druck und Temperatur gleich 32 l.)
a. Wie groß ist der Reifendruck jetzt bei 19o C ?
b. Wie groß ist das Volumen der Luft bei Normaldruck und 19o C, das nachgepumpt werden
muss, damit im Reifen wieder 2,4 bar Überdruck herrschen?
Seite 4 von 11
5. Blitz
Seite 5 von 11
Lösung zu Aufgabe 1:
Ein Blitz deponiert 0,8 Kilowattstunden an Wärme in einer Luftsäule der Länge 250 m und
mit Durchmesser 1 cm. Die Luft habe vor dem Blitz die Temperatur 0o C und Normaldruck
(105 Pa). Luft sei als ideales Gas behandelt. (Molvolumen bei Normaldruck und 0o C: Vm =
22 l, cmv = 20 (kJ)/(Kkmol), Adiabatenkoeffizient κ = 1,4). Wärmeleitung sei vernachlässigt.
a. Die Luftsäule habe zunächst ein konstantes Volumen. Auf welche Temperatur heizt sich
die Luftsäule auf? Welcher Druck stellt sich ein? (Ionisierung vernachlässigt)
b. Die Luftsäule dehnt sich dann adiabatisch auf das hundertfache ihres Volumens aus.
Welche Temperatur stellt sich dann ein?
c. Wie bezeichnet man die durch einen Blitz hervorgerufene plötzliche Druckänderung und
ihre Ausdehnung?
Seite 6 von 11
Seite 7 von 11
Lösung zu Aufgabe 2:
Lösung zu Aufgabe 3:
Seite 8 von 11
Lösung zu Aufgabe 4:
Seite 9 von 11
Lösung zu Aufgabe 5:
Seite 10 von 11
Seite 11 von 11
Name, Matrikelnummer:
1. Elektromagnetische Kugelwelle
Eine elektromagnetische Kugelwelle erzeugt im Abstand von 240 m von der Quelle die
1 V
elektrische Feldstärke E (240m, t ) = 12 ⋅ 10 −7 sin(8,87 ⋅ 10 9 − 1,072 ⋅ 1016 ⋅ t ) . Die Phase der
s
m
Welle ist zum Zeitpunkt t = 0 am Ort der Quelle gleich null.
Klausur Physik 2 am 15.3.01 (alte DPO)
Fachbereich Elektrotechnik und Informatik
a. Wie groß sind die Frequenz, die Wellenlänge und die Phasengeschwindigkeit der Welle?
Zugelassene Hilfsmittel: Zusammenfassung der Vorlesung (maximal 6 Seiten A4 einseitig
oder 3 Seiten A4 doppelseitig), Beiblätter zur Vorlesung Physik 2 im WS99, WS 98 oder WS
97, Taschenrechner (ohne drahtlose Übertragung mit einer Reichweite von größer als 30 cm
wie Funkmodem, IR-Sender)
b. Wie lautet die elektrische Feldstärke im Abstand von 60 m von der Quelle in
Abhängigkeit von der Zeit?
Dauer: 2 Stunden
Maximal erreichbare Punktezahl: 100. Bestanden hat, wer mindestens 50 Punkte erreicht.
Bitte beginnen Sie die Lösung der Aufgabe unbedingt auf dem betreffenden
Aufgabenblatt! Falls Sie weitere Blätter benötigen, müssen diese unbedingt deutlich mit
der Aufgabennummer gekennzeichnet sein.
Bitte kennzeichnen Sie dieses Blatt und alle weiteren, die Sie verwenden, mit Ihrem Namen
und Ihrer Matrikelnummer.
AUFGABE
1.a
1.b
2.a
2.b
2.c
3.a
3.b
4.a
4.b
Summe
MÖGLICHE
ERREICHTE
PUNKTZAHL PUNKTZAHL
16
9
10
10
5
14
11
14
11
100
Seite 1 von 9
Seite 2 von 9
3. Kleinbildkamera
2. Licht im Kunststoffblock
Licht der Wellenlänge 660 nm fällt von links aus dem Vakuum (c0 = 3,0.108 m/s) auf einen
durchsichtigen Kunststoffblock. Im Kunststoff hat das Licht eine Wellenlänge von 550 nm.
Eine Kleinbildkamera (Filmformat 24 mm x 36 mm) hat ein Objektiv der Brennweite 50 mm.
Soll die Entfernungseinstellung von ’ auf den geringsten Objektabstand verändert werden,
so muss das Objektiv um die Strecke 10 mm vom Film weg verschoben werden.
a. Welches ist der geringste Objektabstand, bei dem noch ein scharfes Bild auftritt?
Achtung! Zeichnung ist nicht
maßstäblich!
einfallendes
Licht
b. Welche Objektfläche (senkrecht zur optischen Achse) kann beim geringsten
Objektabstand scharf auf den Film abgebildet werden?
a. Wie groß ist der Grenzwinkel für Totalreflexion beim Austritt von Licht aus dem
Kunststoffblock?
b. Unter welchem Winkel zum Lot muss der Strahl von links auf den Block auftreffen, damit
an der oberen Kante gerade Totalreflexion auftritt (d.h., der Einfallswinkel an der oberen
Fläche gerade gleich dem Grenzwinkel für Totalreflexion ist)?
c. Nun wird der gleiche Versuch mit Licht der Wellenlänge 420 nm gemacht. Treten die
gleichen Winkel auf? Wovon hängt das ab?
Seite 3 von 9
Seite 4 von 9
Lösung zu Aufgabe 1:
4. Erhitztes Gefäß
TD016
Ein zylindrischer luftgefüllter Becher mit einem Innendurchmesser von 12 cm wird bei
Normaldruck von 1,013.105 Pa auf 100o C erhitzt, so dass die Luft im Innern diese
Temperatur angenommen hat. Nun wird das Gefäß mit einem Deckel luftdicht verschlossen,
und man lässt den Becher samt Luft auf 20o C abkühlen. Die Luft sei als ideales Gas
betrachtet.
a. Welcher Druck herrscht dann in dem verschlossenen Becher? Wieviel Prozent des
Normaldrucks sind das?
b. Welche Kraft muss man aufwenden, um den Deckel vom Becher zu heben?
Vernachlässigen Sie die Masse des Deckels. Welche Masse hätte ein Körper mit dieser
Gewichtskraft?
Seite 5 von 9
Lösung zu Aufgabe 2:
Seite 6 von 9
Lösung zu Aufgabe 3:
Seite 7 von 9
Seite 8 von 9
Lösung zu Aufgabe 4:
Name, Matrikelnummer:
Klausur Physik 2 am 20.9.01 (alte DPO)
Fachbereich Elektrotechnik und Informatik
Zugelassene Hilfsmittel: Zusammenfassung der Vorlesung (maximal 6 Seiten A4 einseitig
oder 3 Seiten A4 doppelseitig), Beiblätter zur Vorlesung Physik 2 im WS99, WS 98 oder WS
97, Taschenrechner (ohne drahtlose Übertragung mit einer Reichweite von größer als 30 cm
wie Funkmodem, IR-Sender, Bluetooth)
Dauer: 2 Stunden
Maximal erreichbare Punktezahl: 100. Bestanden hat, wer mindestens 50 Punkte erreicht.
Bitte beginnen Sie die Lösung der Aufgabe unbedingt auf dem betreffenden
Aufgabenblatt! Falls Sie weitere Blätter benötigen, müssen diese unbedingt deutlich mit
der Aufgabennummer gekennzeichnet sein.
Bitte kennzeichnen Sie dieses Blatt und alle weiteren, die Sie verwenden, mit Ihrem Namen
und Ihrer Matrikelnummer.
AUFGABE
1.a
1.b
1.c
2.a
2.b
2.c
3.a
3.b
3.c
4.a
4.b
4.c
Summe
MÖGLICHE
ERREICHTE
PUNKTZAHL PUNKTZAHL
9
10
6
9
9
7
8
9
8
9
8
8
100
Seite 9 von 9
Seite 1 von 10
1. Chip-Lithografie
2. Kombinierte Aufhängung
Bei der Chip-Lithografie wird eine Maske (Objekt) optisch auf die Silicium-Unterlage
projiziert (Bild). Bei dieser Projektion sollen die Strukturen der Maske auf 20% ihrer Größe
verkleinert werden.
a. Skizzieren Sie maßstabsgerecht eine solche Anordnung, bestehend aus Linse, Lichtquelle,
Maske und Silicium-Unterlage. Objekt und Bild können als Pfeile gezeichnet werden, das
abbildende System als Ebene. Tragen Sie Objekt- und Bildweite, sowie Objekt- und
Bildgröße ein.
b. Bei einer Bildweite von 9 mm, wie groß muss im obigen Beispiel dann die Brennweite der
Linse sein?
Betrachten Sie die folgende Aufhängung:
Aluminium
l1
l2
Stahl
l
c. Welche Maßnahmen kann man ergreifen, um die Intensität des Bildes zu erhöhen, ohne
die Stärke der Lichtquelle zu verändern?
Der linke Stab der Länge l1 besteht aus Stahl (Längenausdehnungskoeffizient: 11.10-6 1/K),
der rechte aus Aluminium (Länge l2, Längenausdehnungskoeffizient: 24.10-6 1/K). Die obere
Verbindungsplatte spielt für die Aufgabe keine Rolle.
a. Geben Sie die Änderung der Höhe l bei einer Temperaturänderung um ∆t (∆t in oC) an in
Abhängigkeit der Längen l1, l2, der Ausdehnungskoeffizienten und der
Temperaturänderung.
b. Wenn der Stahlstab 72 mm lang ist, wie lang muss dann der Aluminiumstab sein, damit
sich l mit der Temperatur nicht ändert?
c. Dehnen sich alle Stoffe bei Temperaturerhöhung aus? Falls nein, nennen Sie ein
Gegenbeispiel.
Seite 2 von 10
Seite 3 von 10
3. Expansion
TD017
4. Dualismus Welle-Teilchen, Neutron u. Photon
CO2 mit einer Anfangstemperatur von 360o C expandiert ohne Wärmeaustausch mit der
Umgebung von einem Anfangsvolumen von 0,4 l auf ein Endvolumen von 2,2 l. CO2 sei als
ideales Gas betrachtet. Die molare Wärmekapazität bei konstantem Druck beträgt
36,9 J/(Kmol), die molare Wärmekapazität bei konstantem Volumen 28,6 J/(Kmol).
AK008
a. Ein Neutron (Ruhemasse m = 1,67.10-27 kg) bewegt sich mit einer Geschwindigkeit von
4377 m/s. Wie groß ist die Phasengeschwindigkeit der Materiewelle? (Beachten Sie die
de-Broglie-Gleichungen, die Gesamtenergie des Neutrons berechnet sich zu E = m.c02, m
wie oben, c0 = 2,99.108 m/s, h = 6,63.10-34 Js. Relativistische Effekte treten nicht auf.)
a. Wie bezeichnet man diesen Prozess? Skizzieren Sie den Prozess in einem pV-Diagramm.
b. Wie groß ist die Endtemperatur des Gases beim Volumen 2,2 l in oC?
b. Ein (kohärenter) Neutronenstrahl mit Bedingungen wie bei a. durchlaufe jetzt ein
(atomares) Gitter mit einem Spaltabstand von 0,5 nm. Unter welchem Winkel (in Grad)
tritt das erste Beugungsmaximum auf?
c. Wie ändert sich (qualitativ) die innere Energie des Gases bei diesem Prozess? Wie lautet
für diesen Prozess der 1. Hauptsatz der Thermodynamik? (Bitte schreiben Sie nicht den
Satz aus den Beiblättern ab!)
c. Gegeben seien Photonen mit Energie von jeweils 2,66.10-19 J. Welche Wellenlänge hat
diese Strahlung in einem Medium mit Brechzahl 1,55?
Seite 4 von 10
Lösung zu Aufgabe 1:
Seite 5 von 10
Lösung zu Aufgabe 2:
Seite 6 von 10
Seite 7 von 10
Lösung zu Aufgabe 3:
Lösung zu Aufgabe 4:
Seite 8 von 10
Seite 9 von 10
Name, Matrikelnummer:
Lösung zu Aufgabe 5:
Klausur Physik 2 am 14.3.02 (alte DPO)
Fachbereich Elektrotechnik und Informatik
Zugelassene Hilfsmittel: Zusammenfassung der Vorlesung (maximal 6 Seiten A4 einseitig
oder 3 Seiten A4 doppelseitig), Beiblätter zur Vorlesung Physik 2 im WS99, WS 98 oder WS
97, Taschenrechner (ohne drahtlose Übertragung mit einer Reichweite von größer als 30 cm
wie Funkmodem, IR-Sender, Bluetooth)
Dauer: 2 Stunden
Maximal erreichbare Punktezahl: 100. Bestanden hat, wer mindestens 50 Punkte erreicht.
Bitte beginnen Sie die Lösung der Aufgabe unbedingt auf dem betreffenden
Aufgabenblatt! Falls Sie weitere Blätter benötigen, müssen diese unbedingt deutlich mit
der Aufgabennummer gekennzeichnet sein.
Bitte kennzeichnen Sie dieses Blatt und alle weiteren, die Sie verwenden, mit Ihrem Namen
und Ihrer Matrikelnummer.
AUFGABE
1.a
1.b
1.c
1.d
2.a
2.b
2.c
3.a
3.b
3.c
4.a
4.b
4.c
Summe
Seite 10 von 10
MÖGLICHE
ERREICHTE
PUNKTZAHL PUNKTZAHL
7
6
6
6
9
9
7
10
10
5
10
10
5
100
Seite 1 von 9
1. Baukran mit Seil
2. Motordrehzahl
Von einem Baukran hängt ein Seil herunter. Es führt mit dem daran befestigten Mörtelkübel
in 25 Sekunden 2 Schwingungen aus. Die Differentialgleichung des Pendels lautet:
d 2s
g
= − s (s: Auslenkung, g: Erdbeschleunigung, l: Länge des Pendels)
l
dt 2
a. Zeigen Sie, dass s = A sin(ωt + ϕ ) Lösung der Differentialgleichung ist.
b. Welche Bedeutung haben Ȧ und ij? Beschreibt die Differentialgleichung eine
gedämpfte oder eine ungedämpfte Schwingung?
a. Ein Auto fährt mit einer Geschwindigkeit von 120 km/h unmittelbar an einem
stehenden akustischen Messgerät vorbei. Beim Vorüberfahren ändert sich die vom
Messgerät erfasste Frequenz des vom Auto abgestrahlten Tons um 19,8 Hz. Die
Schallgeschwindigkeit beträgt 340 m/s. Wie groß ist die Drehzahl des Automotors
(gleichgesetzt der Frequenz des abgestrahlten Tons) in Umdrehungen pro Minute?
b. Ein Cabriolet entfernt sich mit einer Geschwindigkeit von 90 km/h von einer
stationären Schallquelle. Die Schallquelle sendet Wellen einer Wellenlänge von 3,4 m
aus. Welche Frequenz hört der Fahrer? (Schallgeschwindigkeit wie bei a.)
c. Warum kann beim Dopplereffekt von elektromagnetischen Wellen nicht zwischen
bewegtem Beobachter und bewegter Quelle unterschieden werden?
c. Wie lang ist das Seil?
d. Welche Geschwindigkeit hat der Mörtelkübel an der tiefsten Stelle, wenn die
maximale (seitliche) Auslenkung 0,5 m beträgt?
Seite 2 von 9
Seite 3 von 9
3. Sonnenbrille
4. Gaspatrone
Eine Gaspatrone zur Erzeugung von Sprudelwasser enthält 100 g des Gases und reicht aus, 50
Literflaschen mit 1,3 bar (1 bar = 105 Pa) des Gases zu füllen. (Die Lösung des Gases im
Wasser wird vernachlässigt.)
a. Schätzen Sie anhand dieser Angaben die Molmasse (Masse pro Mol) des Gases ab.
(Gaskonstante = 8,314 J/(mol.K), Gas als ideal betrachtet)
b. Wieviel Argonatome befinden sich in einem Kubikmeter, wenn man 1 Mol Argon
gleichmäßig auf das Volumen der Erde verteilt?
(Avogadrokonstante = 6,022.1023 mol-1, Teilchen pro Mol)
c. Ist Luft ein ideales Gas? Begründen Sie Ihre Antwort.
Die obige Anzeige stammt aus einem Versandkatalog.
a. Skizzieren und beschreiben Sie die Wirkungsweise dieser Sonnenbrille. Warum sind zwei
Filter notwendig (im Text fälschlich Linsen genannt)?
b. Durch Drehen des Rädchens soll die Intensität des Lichts um den Faktor 4 verkleinert
werden, ausgehend von der maximalen Intensität. Um welchen Winkel werden die Filter
dabei verdreht? Wie verändert sich die Intensität des durchgelassenen Lichts, wenn man
die gesamte Brille dreht?
c. Normales Tageslicht ist teilweise polarisiert. Warum ist das so? Wozu brauchen
Fotografen Polarisationsfilter?
Seite 4 von 9
Seite 5 von 9
Lösung zu Aufgabe 1:
Lösung zu Aufgabe 2:
Seite 6 von 9
Lösung zu Aufgabe 3:
Seite 7 von 9
Lösung zu Aufgabe 4:
Seite 8 von 9
Seite 9 von 9
Herunterladen